Équivalence des normes

Bonjour
Soit( $\mathbb{X} , \Vert . \Vert ) $ un espace de Banach et $p \in \left[ 1, \infty \right[ $
On se donne une fonction localement intégrable $ f \in L_{loc}^{p}(\mathbb{R},\mathbb{X}) $
On définit ainsi les normes

$$\Vert f \Vert_{S_{L}^{p}} = \sup\limits _{x\in \mathbb{R}} \left( \frac{1}{L} \int_{x}^{x+L} \Vert f(t)\Vert ^{p} \right)^{\frac{1}{p}} \ , L >0$$
je cherche à montrer que ces normes sont équivalentes
C'est à dire $\forall L_{1} , L_{2} \in \mathbb{R}_{+} \exists k_{1} , k_{2} \in \mathbb{R}_{+} k_{1}\Vert f \Vert_{S^{p}_{L_{1}}} \leq \Vert f \Vert_{S^{p}_{L_{2}}} \leq k_{2}\Vert f \Vert_{S^{p}_{L_{1}}}$
Comme L est dans le bord d'intégrale j'ai pensé à faire un changement de variable mais ça ne marche pas
Quelqu'un a-t-il une idée ? Merci d'avance
Cordialement

Réponses

  • Etant donné un segment de longueur $L_{2}>L_{1}$ combien faut-il ( au minimum) de segments de longueur $L_{1}$ pour le recouvrir?
  • Je n'arrive pas à trouver la réponse ça dépend aux longueurs $ L_{1}$ et $L _{2}$.
  • Je pense que les $L_i$ sont STRICTEMENT positifs.

    Commence par le cas où $L_2/L_1$ est entier. Et si tu ne vois pas, commence carrément par $L_2=2L_1$.

    Tiens des normes de Stepanov, ça sent les fonctions presque-périodiques à plein nez.
  • Remarque juste que $$[x,x+L_ {2}]\subset \bigcup_{k=0}^{\big\lfloor \frac{L_{2}}{L_{1}} \big\rfloor +1} [x+kL_{1},x+(k+1)L_{1}].
    $$ Conclus à partir de cette inclusion !
  • Oui L >0
  • Merci pour l'indication mais je n'arrive pas à le montrer
  • Est-ce qu'on peut se débarrasser des $\frac{1}{L}$, déjà ?
    On poserait simplement : $$
    \|f\|_{S_{L}^{p}} = \sup_{x\in \mathbb{R}} \left( \int_{x}^{x+L} \Vert f(t)\Vert ^{p} \right)^{\frac{1}{p}}
    $$ Comme ça, elles croissent avec $L$.
    Il ne reste plus qu'à les majorer en fonction de : $\displaystyle\|f\|_{S_{1}^{p}} = \sup_{x\in \mathbb{R}} \left( \int_{x}^{x+1} \Vert f(t)\Vert ^{p} \right)^{\frac{1}{p}}$
    La croissance, c'est vraiment fort. Il suffit par exemple de le faire pour $L$ entier (ou une puissance de 2, ou n'importe quelle suite qui $\to \infty$...)
  • Pour te débloquer, le cas $L_1 = 2L_2$
    \begin{align*}
    \int_x^{x+L_1} \| f(t) \|^p dt &=
    \int_x^{x+L_2} \| f(t) \|^p dt +\int_{(x+L_2)}^{(x+L_2) + L_2} \| f(t) \|^p dt\\
    &\leq \sup_{x\in \mathbb{R}} \int_x^{x+L_2} \| f(t) \|^p dt + \sup_{x\in \mathbb{R}}
    \int_{(x+L_2)}^{(x+L_2) + L_2} \| f(t) \|^p dt\\
    &\leq 2 \sup_{x\in \mathbb{R}} \int_x^{x+L_2} \| f(t) \|^p dt
    \end{align*}
    Tu vois comment finir ?
  • Oui merci
  • Demandé en MP (on peut se tutoyer!)

    On part de : \[\|f\|_{L,P} = \left|\frac{1}{L} \cdot \sup_{x\in\R} \int_{x}^{x+L} \|f(t)\|^p dt \right|^\frac{1}{p}
    \] Alors la norme suivante est équivalente : \[
    \|f\|'_{L,P} = \left|\sup_{x\in\R} \int_{x}^{x+L} \|f(t)\|^p dt \right|^\frac{1}{p}
    \] En effet, on a : \[\|f\|_{L,P} = \frac{1}{L^\frac{1}{p}} \cdot \|f\|'_{L,P}.
    \] Pour toute la suite de l'exercice (équivalence pour des valeurs de $L$ différentes), si :
    $N_1$ et $N_2$ sont équivalentes,
    $N_2$ et $N_3$ sont équivalentes,
    alors :
    $N_1$ et $N_3$ sont équivalentes.
  • Merci
Connectez-vous ou Inscrivez-vous pour répondre.